Preguntas matemáticas abiertas para las que realmente no tenemos idea de cuál es la respuesta

No hay escasez de problemas abiertos en matemáticas . Si bien una prueba formal para cualquiera de ellos sigue siendo difícil de alcanzar, con las preguntas de "sí / no" entre ellas, los matemáticos generalmente no trabajan en ambas direcciones, sino que tienen una idea bastante clara de cuál debería ser la respuesta . Conjeturas famosas como las de Riemann y Collatz están respaldadas por algunas heurísticas muy convincentes, lo que lleva a los matemáticos a creer en su validez con tanta fuerza que escriben artículos basados ​​en la suposición de que son verdaderas. Para otros problemas abiertos como PAG contra norte PAG , un lado ( PAG = norte PAG en este caso) suele considerarse tan poco probable que casi nadie trabaje seriamente en ello. Por supuesto, cada vez que se adjunta una "conjetura" a una pregunta abierta que ya implica que se prefiere una respuesta sobre la otra, la gente no hace conjeturas. A y ¬ A simultáneamente.

¿Existen preguntas matemáticas abiertas con una respuesta de sí/no para las que no tenemos una buena razón para asumir una u otra, para las cuales realmente no tenemos ni idea de cuál podría ser la verdadera respuesta?

Tal vez hay infinitos números primos de Fermat.
De Wikipedia: "El siguiente argumento heurístico sugiere que solo hay un número finito de números primos de Fermat..."
lo que el o no ζ ( 3 ) (y valores zeta enteros impares en general) tiene una expresión de forma cerrada en términos de constantes fundamentales más básicas, como π o registro 2 , me viene a la mente. Hay muy buenas razones para creer que sí, así como muy buenas razones para creer que no. Lo que no significa una buena razón para ninguno de los dos...
@DavidH: Mi dinero está en el lado "no".
¿Qué pasa con los valores de números de Ramsey suficientemente grandes? Ver, por ejemplo, la cita de Erdős sobre cómo es mejor que intentemos destruir a los extraterrestres omnicidas que quieren saber. R ( 6 , 6 ) que tratar de calcularlo para ellos...
@Micah: Tenga en cuenta que estoy preguntando específicamente sobre preguntas de sí/no (porque estaba pensando en el mismo ejemplo cuando escribí la pregunta).
¿Todos los números primos aparecen en la sucesión Euclid-Mullin?
@Micah ¿Hay alguna buena razón para creer que límite norte R ( norte , norte ) 1 / norte existe (o no existe)? O que el límite existe y es igual a 2 ?
Esto está fuera de mi experiencia, por lo que lo pondré como un comentario en lugar de una respuesta. No se sabe si el grupo de Burnside B ( 2 , 5 ) es finito algunos grupos B ( metro , norte ) son, algunos no lo son. No sé si los expertos tienen un consenso sobre B ( 2 , 5 ) . Para obtener algunos detalles, consulte www-history.mcs.st-and.ac.uk/HistTopics/Burnside_problem.html
Si no se ha probado o refutado, entonces realmente no sabemos la respuesta. Puede haber una preponderancia de opiniones y prejuicios a favor de un lado o del otro, pero las opiniones son irrelevantes en matemáticas. Incluso las opiniones de los "expertos", cuya pericia no se extiende a saber probar o refutar la afirmación en cuestión.
¿Hay alguna buena razón para creer que la conjetura de Frankl sobre las familias cerradas por unión es verdadera (o falsa)? ¿Hay alguna buena razón para creer que la conjetura de Vizing sobre el número cromático del borde de la lista (elección del borde) de los gráficos es verdadera (o falsa)?
@bof, estuve tentado de sugerir la conjetura de Frankl. Supongo que la evidencia a favor es que nadie ha encontrado un contraejemplo, y se ha probado que no hay pequeños contraejemplos, pero, en cuanto a B ( 2 , 5 ) , no sé si los expertos tienen un consenso.
@GerryMyerson Y la evidencia en contra es que nadie ha encontrado una prueba. No soy un experto, pero la última vez que lo investigué, habían demostrado que "no había pequeños contraejemplos" solo para un valor realmente pequeño de "pequeño".
Para que conste, Knuth dijo que cree que P=NP es más plausible, en una entrevista reciente: informit.com/articles/article.aspx?p=2213858
@ypercube: Esto me sorprende por decir lo menos. ¿Tienes una referencia/enlace?
@pew Agregó el enlace de arriba. Pero él piensa que si encontramos una prueba, probablemente será una existencial, no un algoritmo real, útil en aplicaciones prácticas.

Respuestas (23)

En 4 dimensiones, es una pregunta abierta si hay estructuras suaves exóticas en las 4 esferas.

En este sentido, ¿existe una estructura compleja en S 6 ?
@MikeMiller Si este documento es correcto math.bme.hu/~etesi/s6-spontan.pdf
Hay una gran cantidad de artículos que afirman haber resuelto el problema (ya sea positiva o negativamente), y se ha demostrado que muchos tienen errores. Mi impresión es que todavía se acepta generalmente que el problema está abierto.
De hecho, este es un gran ejemplo. No creo haber escuchado a ningún experto en el campo siquiera adivinar si hay una, un número finito o un número infinito de estructuras exóticas en S 4 .
Scorpan menciona que no hay lisos compactos simplemente conectados 4 -variedades conocidas por admitir solo un número finito de estructuras suaves.

Un ejemplo más o menos elemental que me gusta mucho es la conjetura de Erdős sobre las progresiones aritméticas , que afirma lo siguiente:

Si para algún conjunto S norte la suma

s S 1 s
diverge, entonces S contiene progresiones aritméticas arbitrariamente largas.

Nunca he visto un argumento heurístico de una forma u otra; creo que el resultado conocido más fuerte, a partir de ahora, es el teorema de Szemerédi , que, más o menos, establece que si la densidad asintótica más baja de S es positivo (es decir, hay un número infinito de norte tal que | [ 1 , norte ] S | > norte ε ), entonces contiene progresiones aritméticas arbitrariamente largas. También está el teorema de Green-Tao , que es un caso especial de la conjetura, dado que los números primos tienen progresiones aritméticas arbitrariamente largas (y, de hecho, también establece el hecho para una clase más grande de conjuntos).

Sin embargo, ninguno de estos sugiere que el resultado se mantenga en general. Es tentador creer que es cierto, porque sería un teorema tan hermoso, pero no hay mucho que lo respalde; en realidad no está claro por qué la suma de los recíprocos divergentes tendría algo que ver con las progresiones aritméticas. Aún así, no hay ejemplos obvios de dónde falla, por lo que tampoco es difícil argumentar en su contra.

"... realmente no está claro por qué la suma de los recíprocos divergentes tendría algo que ver con las progresiones aritméticas" parece demasiado derrotista. La intuición es clara: si una colección de números naturales es lo suficientemente grande, no puede evitar ciertos patrones. Ese es uno de los tipos básicos de lecciones que aprendes en la teoría de Ramsey, y tanto el teorema de Szemeredi como el teorema de Green-Tao son excelentes ejemplos de esa filosofía en acción. La suma de los recíprocos divergentes es un candidato decente para "lo suficientemente grande"; seguramente el ejemplo de los números primos es lo que motivó a Erdos a conjeturar el resultado...
... Entonces, ¿por qué no apegarse a la conjetura de Erdos? No es como si simplemente escribiera al azar esta conjetura. La gente conjetura cosas por razones.
@Qiaochu Lo que quiero decir es que hay una variedad de funciones que podrían satisfacer " s S F ( s ) es grande implica S contiene un AP arbitrariamente largo" - el teorema de Szemeredi proporciona un F que funciona, que es suficiente para un límite superior. Y, sí, la suma de los recíprocos de los números primos diverge, pero también lo hace la suma de los números primos. 1 norte registro ( registro ( norte ) ) , y, que yo sepa, no tenemos ninguna razón para suponer que este es un límite inferior menos razonable que 1 norte . Qué distingue 1 norte es que es, más o menos, la función decreciente más lenta para la que se podría cumplir el enunciado...
...desde que resumimos 1 norte 1 ε es casi seguro que va a dar conjuntos que tienen sumas divergentes, pero AP no lo suficientemente largos (ya que los conjuntos solo necesitan O ( X 1 1 ε ) elementos). Entonces, 1 norte se deja en el límite, lo que significa que la conjetura es una pregunta muy razonable, ya que 1 norte es el primer caso abierto que tenemos. Pero "pregunta razonable" está lejos de "la respuesta es sí"
La formulación con la suma era solo una forma pegadiza de expresar las cosas. De todos modos, esta no es una respuesta, ya que hay muy buenas razones para creerlo. El caso de 3-AP está, en cierto sentido, bastante cerca de ser resuelto por los resultados recientes de Tom Sanders.
@quid Precisamente, ¿cómo sugiere definir el problema sin la suma? Me sorprende que incluso los resultados asintóticos estrictos podrían no establecer la conjetura, principalmente porque, fuera del dominio de cada subconjunto de norte , las que se comportan lo suficientemente bien como para ser efectivamente descritas por ideas asintóticas me parecerían bastante pequeñas y, además, dado cualquier conjunto contable de funciones que represente la densidad de un conjunto grande, podemos construir otro, creciendo más lentamente que cualquier otro. de los demás: no veo cómo los resultados asintóticos pueden manejar eso.
@quid Pero eso sugiere que deberíamos poder demostrar que, si A es un subconjunto de | { 1 , , norte } | sin AP-3, entonces probablemente sostiene que
| A | = O ( norte registro norte ( registro registro norte ) 2
pero no tenemos ninguna razón para creer que esto es cierto, ¿y si el límite es O ( norte registro norte registro registro norte ( registro registro registro norte ) 2 , ¿o algo así? O qué tal si podemos conseguir A en O ( norte registro norte registro registro norte ( registro registro norte ) 2 no importa cuántos troncos pongamos? Supongo que tiene razón en que la asintótica es suficiente si el límite es suelto, por lo que el interés de la declaración es si se mantiene y es estrecho .
Tu último comentario parece incompleto. El punto es que las creencias para los límites reales de la cardinalidad son tales que la suma convergerá. se desconoce cuáles son las asintóticas realmente precisas; sin embargo, vea mi respuesta para algo relacionado. Todavía se cree que los conjuntos sin kAP son lo suficientemente pequeños como para que la suma converja.

Creo que si el grupo de Thompson F es susceptible es tal pregunta. El paper/artículo " WHAT is... Thompson's Group " menciona que en una conferencia dedicada al grupo hubo una encuesta en la que 12 dijeron que sí y 12 dijeron que no. De hecho, hay documentos que afirman (al menos en ese momento) tener pruebas para ambos lados. Aquí hay algunas publicaciones para tener una idea de la "controversia": 1 , 2 , 3 .

Me parece que cada vez que alguien pronuncia las palabras "grupo de Thompson" (¡no importa de cuál esté discutiendo en realidad!) alguien más le preguntará sobre la posibilidad de F . Creo que este problema es tan interesante simplemente por la cantidad de ataques falsos (incluso por parte de Justin Moore, quien habló en el ICM), y porque esos ataques afirman resultados opuestos.
Se acepta, ya que esta respuesta no solo brinda un problema abierto, sino también evidencia de que cualquiera de los lados podría ser cierto, según el conocimiento actual.

El décimo problema de Hilbert terminado q /Conjetura de Mazur. Estos son dos problemas abiertos que apuntan en direcciones opuestas, y creo que los expertos realmente no están seguros de qué manera adivinar.

El décimo problema de Hilbert terminado q ¿Existe algún algoritmo que, dada una colección de ecuaciones polinómicas con coeficientes racionales, tengan una solución racional?

El problema está abierto. Aquí hay heurísticas en cada sentido. Por "no".

  • Las ecuaciones diofánticas individuales son realmente difíciles. Piense en cuántos matemáticos trabajaron para probar X norte + y norte = 1 no tiene más soluciones que ( 0 , 1 ) y ( 1 , 0 ) para varios valores de norte . ¿Es realmente plausible que todo su trabajo pueda reducirse a ejecutar un algoritmo?

  • No existe tal algoritmo Z . ( Matiyasevich-Robinson-Davis-Putnam )

Para "sí":

  • Existen poderosos teoremas y conjeturas acerca de las ecuaciones diofánticas que tienen un número finito de soluciones. Por ejemplo, la conjetura de Mordell (ahora el teorema de Falting ) nos dice inmediatamente que hay un número finito de puntos racionales en X norte + y norte = 1 para cualquier dado norte . Si se probara la conjetura de Bombieri-Lang , que no parece imposible, tendríamos herramientas mucho más potentes. Y aunque finito no es lo mismo que cero, hemos desarrollado muchas herramientas para encontrar esas soluciones finitas en muchos casos. Vea las notas del curso de Bjorn Poonen para una encuesta.

Pero aquí está lo realmente frustrante. Suponga que cree que la respuesta es "no". Entonces probablemente quiera probar que puede codificar el problema de detención como una pregunta sobre ecuaciones diofánticas (así fue como se probó MDRP), o bien codificar la resolución de ecuaciones diofánticas sobre Z en resolver ecuaciones diofánticas sobre q . Para hacer esto, presumiblemente escribirías alguna ecuación diofántica cuyas soluciones se parecieran a los estados de una máquina de Turing universal, o se parecieran a Z . En cualquier caso, probablemente tendría infinitas soluciones, repartidas discretamente. Y así te encuentras

Conjetura de Mazur Dada cualquier colección de ecuaciones polinómicas sobre q en norte -variables, vamos X ( q ) sea ​​el conjunto de sus soluciones y sea X ( q ) ¯ Sea el cierre topológico de X ( q ) en R norte . Entonces X ( q ) ¯ tiene un número finito de componentes conexas.

Entonces, la dificultad general de las ecuaciones diofánticas lleva a uno a imaginar que el problema no tiene solución, pero la conjetura de Mazur bloquea la ruta más plausible para demostrar que no tiene solución. Por supuesto, uno puede imaginar que las ecuaciones diofánticas sobre q son irresolubles y, sin embargo, no pueden codificar una máquina de Turing. Creo que no está claro si la mayoría de los problemas irresolubles son irresolubles porque son equivalentes al problema de la detención, o si ese es (esencialmente) el único tipo de problema que sabemos cómo demostrar que es irresoluble.

¡Esto es realmente convincente! +1
"¿Es realmente plausible que todo su trabajo pueda reducirse a ejecutar un algoritmo?" Debido a que finalmente se encontró una prueba, la habría encontrado un algoritmo que intentaba y verificaba todas las pruebas posibles, ¿verdad?
"Creo que no está claro si la mayoría de los problemas irresolubles son irresolubles porque son equivalentes al problema de la detención, o si ese es (esencialmente) el único tipo de problema que sabemos cómo demostrar que es irresoluble". - Tengo entendido que puede probar que la función del castor ocupado no es computable sin usar el problema de detención, pero podría estar equivocado, y sé que hay pruebas que lo usan.
@Kevin De hecho, sé mucho sobre esto. Estoy dejando un comentario para poder encontrar este hilo más tarde cuando tenga tiempo de escribir algo.
No recuerdo si son todos, algunos o solo algunos, pero sí recuerdo que es muy posible que los problemas irresolubles también tengan la agonizante propiedad de que ni siquiera se puede probar su irresoluble.
Tenga en cuenta que puede haber problemas que no son computables, pero para los cuales un oráculo no permite resolver el problema de la detención; como un ejemplo simple, el problema de decisión para un subconjunto aleatorio de norte es uno de esos Para obtener más detalles, consulte en.wikipedia.org/wiki/Turing_degree .

En la teoría de sistemas dinámicos, los problemas que involucran ciclos límite en general son siempre muy difíciles. La segunda parte del decimosexto problema de Hilbert es mi "favorito" personal. El límite superior para el número de ciclos límite de campos vectoriales polinómicos planos de grado norte permanece sin resolver para cualquier norte > 1 . Por ejemplo, ¿pueden los campos vectoriales del plano cuadrático ( norte = 2 ) tienen más de cuatro ciclos límite? Puede ser extremadamente complicado encontrar un sistema cuadrático con cinco ciclos límite, pero realmente no tenemos ni idea. En la década de 1950, los matemáticos afirmaron que los sistemas cuadráticos tienen un máximo de tres ciclos límite y muchos otros matemáticos lo confirmaron, pero se demostró que era incorrecto cuando se encontró un sistema cuadrático con cuatro ciclos límite. Para más detalles, puedes consultar este artículo .

Por lo que puedo decir, ni la existencia ni la inexistencia del Gráfico de Moore de grado 57 y diámetro 2 está fuertemente atestiguada. La mayor parte del trabajo hasta la fecha sobre el tema gira en torno a las diversas propiedades que un gráfico de este tipo (si existiera) debe o no debe poseer, pero ninguno de estos parece dar una indicación sólida para inclinarse hacia un lado o hacia el otro. Además, los encuestados en una encuesta en esta publicación de blog de 2009 parecen estar divididos de manera bastante uniforme.

La conjetura suave de Poincaré en la dimensión 4 ya se ha mencionado, así que mencionaré el problema suave de Schönflies en esa dimensión. La pregunta es si existe un difeomorfismo de S 4 tomando cualquier copia incrustada sin problemas de S 3 en S 4 al ecuatorial estándar S 3 S 4 . Esto es cierto en todas las demás dimensiones, pero 4 es una dimensión tan inusual que es difícil especular cuál es la respuesta en este caso.

Desde una perspectiva teórica de números, hay algunos problemas famosos relacionados con los rangos de las curvas elípticas, que gran parte de la investigación moderna en el área está orientada a resolver. Por ejemplo, Manjul Bhargava recibió recientemente la medalla Fields en parte por su trabajo sobre la delimitación de rangos promedio de curvas elípticas (y por demostrar que la conjetura de Birch y Swinnerton Dyer es cierta para porcentajes cada vez mayores de curvas elípticas).

Para describir algunos de los resultados: una curva elíptica sobre q es una curva proyectiva racional suave de género 1 con un punto racional, o en términos menos aterradores, el conjunto de soluciones a una ecuación que se parece a

mi ( q ) = { ( X , y ) q 2 : y 2 = X 3 + a X + b }
dónde a , b q . Es un hecho que cualquier conjunto de este tipo forma un grupo abeliano finitamente generado, por lo que según el teorema de estructura para tales objetos, el grupo de puntos racionales es
mi ( q ) Z r + Δ ,
dónde Δ es un grupo finito. Ahora, tenemos descripciones completas de lo que este grupo Δ puede ser - un Teorema de Mazur lo limita a una pequeña lista finita de grupos finitos de tamaño menor que 12. Sin embargo, los valores de r son mucho más misteriosos. Definimos el rango de mi ser esto r = r ( mi ) .

Ahora, sabemos bastante acerca de r - por ejemplo, en el "100%" de los casos el rango es 0 o 1 (donde aquí "100%" se usa en el sentido probabilístico, sin significar que cada curva elíptica tiene rango 0 o 1 !). También está la conjetura de Birch y Swinnerton Dyer (BSD) , que es uno de los problemas muy abiertos que mencionas que nadie tiene idea de cómo probar, pero que la mayoría de la gente cree. Relaciona el rango de la curva elíptica con el orden de desaparición de sus L -función en 1. Quizás la heurística más fuerte para esto es que se ha probado en ciertos casos especiales, así como en el trabajo de Bhargava. Gran parte de la investigación de la teoría de números moderna se dirige hacia BSD, y es uno de los famosos problemas de Millenium.

Sin embargo, con lo que no tenemos mucha intuición es:

Pregunta: ¿Están los rangos de las curvas elípticas sobre q ¿encerrado? Es decir, ¿hay alguna R tal que para cualquier curva elíptica mi / q , tenemos r ( mi ) R ?

Desde el año pasado, era muy abierto: había heurísticas sueltas en ambos sentidos. El rango más alto que hemos encontrado hasta ahora es una curva con un rango de al menos 28, debido a Elkies, que ha sido el poseedor del récord durante mucho tiempo. Como mencioné antes, Bhargava ha demostrado que el rango promedio está limitado por al menos 1,5, y esto fue suficiente para ganar una medalla Fields.

Sin embargo, habiendo dicho todo eso, creo que recientemente ha habido algo de entusiasmo con algunas heurísticas más fuertes que se inclinan hacia el límite del rango. No sé lo suficiente sobre estas heurísticas para comentar más, pero hay más información aquí: http://quomodocumque.wordpress.com/2014/07/20/are-ranks-bounded/

¿Hay alguna conexión aquí con ECC? (En otras palabras, ¿existen implicaciones sobre las curvas elípticas sobre F q ?)
Respuesta corta: no estoy seguro. Respuesta más larga: pregunté en la reunión de mi grupo departamental si había algún resultado que pudiera derivarse de esto, y el consenso parecía no serlo; es el resultado de una naturaleza más independiente, ¡y tener una respuesta definitiva no mejoraría tanto nuestra comprensión con los métodos actuales! Por supuesto, como nadie sabe cuál podría ser la respuesta, es poco probable que se haya realizado algún trabajo asumiendo que los rangos están acotados (o ilimitados), por lo que puede ser difícil saber qué podría derivar de una prueba de cualquier manera.

Un problema abierto fácil de entender implica el primer contraejemplo de la conjetura de la suma de potencias de Euler :

P : ¿ X 1 5 + X 2 5 + X 3 5 + X 4 5 + X 5 5 = 0 ¿ Tiene infinitas soluciones enteras primitivas distintas de cero?

( Primitivo siendo el X i no tienen factor común.) Hasta ahora solo se conocen tres y nadie ha dado un buen argumento heurístico de que la lista es finita, o si hay infinitamente muchos. Hay restricciones congruentes interesantes en el X i .

Más generalmente,

P : Para impares k > 3 , hace X 1 k + X 2 k + + X k k = 0 ¿ Tiene infinitas soluciones enteras primitivas distintas de cero?

No creo que nadie tenga una idea clara de si existen soluciones clásicas a la ecuación de Navier-Stokes. http://www.claymath.org/millenium-problems/navier%E2%80%93stokes-ecuación

La mayoría de los intentos se han centrado en tratar de demostrar que es cierto. Sin embargo, Leray dio una sugerencia para buscar un contraejemplo. Más tarde se demostró que su contraejemplo propuesto nunca funcionaría: J. Nečas, M. Růžička y V. Šverák, On Leray's self-similar solutions of the Navier-Stokes ecuations, Acta Mathematica, 1996, Volumen 176, Número 2, págs. 283-294. Sin embargo, el hecho de que se hayan propuesto contraejemplos sugiere que es razonable pensar que la conjetura es falsa.

Mi punto de vista (ingenuo, no especialista). Como sé, las ecuaciones NS se deducen de una hipótesis falsa : que el fluido es un medio continuo. ¿Es realmente razonable esperar soluciones físicamente realistas en cualquier caso? Ejemplo análogo: singularidades en la gravedad clásica.
Creo que no hay una razón convincente por la que NS deba modelar situaciones realistas. Pero escuché de alguien que hizo comparaciones de experimentos y números que funciona sorprendentemente bien, incluso cuando hay una gran cantidad de turbulencia. (Estaba considerando el flujo de aire sobre los automóviles y encontró fenómenos extraños que ocurrieron tanto en los experimentos como en los números).
Si la regularidad de la NS resulta ser cierta, creo que sugeriría fuertemente que la hipótesis del continuo es una aproximación válida a las partículas del fluido que son muy pequeñas.

Creo que una respuesta adecuada a esta pregunta son ejemplos de preguntas donde la evidencia numérica es extremadamente difícil de obtener. Entonces, por ejemplo, no sabemos nada interesante sobre la conjetura de Collatz, pero al menos sabemos que es cierta para una gran cantidad de casos.

Como ejemplo de algo que no sabemos en absoluto, considere S norte el grupo simétrico, y definir s i ser la transposición adyacente ( i , i + 1 ) . Entonces para una permutación π S norte , definió una descomposición reducida de π ser un producto de longitud mínima de transposiciones que le da π . por ejemplo si π = 4321 entonces w = s 1 s 2 s 1 s 3 s 2 s 1 es una descomposición reducida de π .

Es fácil ver que la longitud mínima de la descomposición reducida es el número de inversiones en π . Por otro lado, la cuestión de cuántas descomposiciones reducidas distintas R ( π ) hay de π es una pregunta ridículamente complicada. Sabemos la respuesta para las permutaciones que tienen patrones particulares (falta de) como las permutaciones vexellarias, en particular la permutación inversa ( norte , norte 1 , , 1 ) , que tiene F norte descomposiciones reducidas, donde F norte es el número de escaleras en forma de cuadros jóvenes de forma ( norte 1 , norte 2 , , 1 ) . Para cualquier otra permutación no trivial, esencialmente no se sabe nada. Para norte = 7 , el número de descomposiciones reducidas para la permutación inversa excede el número de átomos en el universo conocido. Surge una dificultad similar para prácticamente cualquier permutación no trivial. Ni siquiera se puede obtener una estimación del orden de magnitud.

Hacer todas las variedades compactas y lisas de dimensión 5 admitir las métricas de Einstein ?

(Una métrica de Einstein es una métrica de Riemann con curvatura de Ricci constante).

Se puede encontrar una lista de problemas abiertos fundamentales en geometría diferencial y análisis geométrico al final de la excelente encuesta de Yau, Revisión de geometría y análisis . Fue escrito en 2000, por lo que es muy actual.

Aparte: algunos problemas que no encajan en el proyecto de ley (en que hay evidencia de una forma u otra o en que escucho las mismas conjeturas), pero que son interesantes de todos modos son:

  • ¿Admite la 6-esfera una estructura integrable casi compleja?

  • Conjetura de Hopf : ¿Tiene S 2 × S 2 admitir una métrica con curvatura seccional positiva?

  • Conjetura de Chern : ¿Toda variedad afín compacta tiene una característica de Euler que se desvanece?

¿Cuáles son las conjeturas/evidencia de tus últimos tres? No he escuchado conjeturas de ninguna manera antes.
@MikeMiller: (1) Cualquier estructura casi compleja en S 6 tendría que ser incompatible tanto con la métrica estándar ( LeBrun ) como con la estructura simpléctica estándar ( Bryant-Chern ). (3) La conjetura de Chern es cierta para variedades afines completas . (2) Recuerdo haber escuchado las mismas conjeturas para la conjetura de Hopf en un punto (por parte de un par de expertos), pero honestamente no las recuerdo y no me aventuraré a hacer ninguna.

¿ Existe un grupo de torsión infinito finitamente presentado ?

Un grupo de torsión es un grupo donde cada elemento tiene un orden finito. El problema de Burnside (1902) preguntó si existe un grupo de torsión infinito generado finitamente . Este grupo de exponente ilimitado fue construido por Golod y Shafarevich en 1964, mientras que Novikov y Adian lo hicieron para exponente acotado en 1968. Ol'shanskii construyó grupos infinitos generados finitamente, todos cuyos subgrupos propios no triviales son cíclicos de orden a prima fija pag (Grupos "monstruo Tarski"). Sin embargo, todos estos ejemplos son finitamente generados pero no finitamente presentables . La cuestión de si existe o no un ejemplo finitamente presentado sigue abierta. Aparentemente, Rips dio un posible método para construir un grupo de este tipo, pero Ol'shanskii y Sapir giraron su manija sin éxito ( referencia ).

Vale la pena mencionar que Efim Zelmanov recibió una medalla de campo por un problema relacionado, llamado problema de Burnside restringido .

He tenido la impresión de que la mayoría piensa que debería haber un grupo de este tipo, solo que con la tecnología actual no tenemos mucha idea para construir un grupo de este tipo (o en realidad llevar a cabo las ideas que podrían construir un grupo de este tipo) . ¿Hay teóricos de grupos que creen seriamente que no existe tal grupo, o alguna razón para creer que no existe? ¿Son sus opiniones las mismas para el caso de exponentes acotados y no acotados?
Mi impresión, como la de @PaulPlummer, fue que se cree comúnmente que existe un grupo así; es solo que no sabemos cómo construir uno, y los métodos que tenemos son bastante delicados y complicados. (Recuerdo una de esas pruebas de existencia de un seminario que implicó verificar una lista de lavandería de más de cien propiedades).
@PaulPlummer Si no recuerdo mal, mi punto se basó en la oración "Aparentemente, Rips dio un posible método para construir un grupo de este tipo, pero Ol'shanskii y Sapir giraron su manija en vano" y el enlace asociado. El enlace reafirma su impresión de que "la mayoría piensa que debería haber un grupo así", y da una "idea general" para construir dicho grupo. Sin embargo, de memoria, entre esa publicación (2011) y mi publicación (2014) Ol'shanskii y Sapir hicieron que esta idea "funcionara" pero, como señala la oración, no fue lo suficientemente buena. Sin embargo, mis fechas pueden ser incorrectas (no he vinculado al periódico).
He tenido una búsqueda rápida del artículo de Ol'shanskii y Sapir, y parece que no han escrito un artículo juntos desde 2006 fuente . Así que... no estoy exactamente seguro de lo que estaba pensando. Lo siento.

La existencia de planos finitos proyectivos . Todos los ejemplos conocidos tienen potencia de orden primo. Cita:

La existencia de planos proyectivos finitos de otros órdenes es una cuestión abierta. La única restricción general conocida sobre el orden es el teorema de Bruck-Ryser-Chowla de que si el orden norte es congruente con 1 o 2 modificación 4, debe ser la suma de dos cuadrados. esto descarta norte = 6 . el siguiente caso norte = 10 ha sido descartado por cálculos informáticos masivos. No se sabe nada más; en particular, la cuestión de si existe un plano de orden proyectivo finito norte = 12 todavía está abierto.
Este es un problema realmente genial, pero no estoy seguro de que se aplique. ¿No cree la mayoría de la gente que esto resultará ser falso? (es decir, ¿que todos los planos proyectivos tienen un orden de potencia principal?)
@AlexanderGruber, ¿hay algún argumento heurístico que favorezca la conjetura?
@AlexanderGruber: Nunca escuché un argumento heurístico convincente a favor o en contra de la existencia de planos proyectivos de orden de potencia no principal. Quizás la intuición en contra es similar a la de la respuesta de Chris Godsil a una pregunta que le hice una vez sobre los biplanos , que son diseños simétricos con λ = 2 (los planos proyectivos son λ = 1 ).
Hay una conjetura relacionada con los planos proyectivos. Los planos desarguesianos son aquellos en los que se cumple el teorema de Desargues, que son los derivados de identificar puntos con rectas que pasan por el origen en un espacio tridimensional sobre un campo finito. Para cada orden primo, el único plano conocido actualmente es el desarguesiano y se conjetura que este es el único que existe. En otros órdenes de potencia principales, se conocen muchos planos no desarguesianos. Si los planos no desarguesianos son de algún modo deformaciones de los desarguesianos, y si tal deformación no es posible en prime...
... orden, eso explicaría lo que se sabe hasta ahora, pero nunca he escuchado ningún argumento en este sentido. Desde otro punto de vista, uno podría tomar la existencia de planos no desarguesianos como una razón para ser optimista sobre la existencia de planos en órdenes de potencia no primaria.

La primera prueba que mucha gente aprende es que hay infinitos números primos. (Si no es el primero, a menudo es el segundo después del hecho de que 2 es irracional).

Dirichlet consideró una generalización natural de esto, quien demostró que mientras la progresión aritmética a , a + d , a + 2 d , a + 3 d , . . . no tiene una razón trivial para no tener muchos números primos, entonces de hecho contiene infinitos números primos. Esto se conoce como el Teorema de Dirichlet sobre los números primos en las progresiones aritméticas . Sorprendentemente, si hay infinitos números primos, entonces también se sabe que la secuencia tiene asintóticamente 1 / φ ( d ) de todos los números primos, donde φ ( d ) es el número de números hasta d que son relativamente primos para d . En otras palabras, toda progresión aritmética no trivial tiene exactamente el mismo porcentaje de números primos, una especie de teorema de equidistribución.

La siguiente generalización natural es considerar polinomios superiores, como los polinomios cuadráticos. (Por el momento, llame a un polinomio cuadrático si es de la forma a X 2 + b X + C con a 0 ). ¿Es cierto el análogo? ¿Podemos predecir la distribución? De hecho, no hemos encontrado un solo polinomio cuadrático que tome infinitos números primos (ni ningún polinomio de grado > 1). Ni siquiera hemos tenido éxito demostrando que X 2 + 1 toma infinitos números primos, ni tenemos idea de cómo.

Profundizando un poco más, es posible conjeturar densidades utilizando el método del círculo o sus variantes, incluso para polinomios de mayor grado. Pero no tenemos idea de cómo probarlos.

En resumen, es X 2 + 1 cebar infinitamente a menudo?

¿Hay alguien que realmente crea que no es primo infinitamente a menudo? Como dijiste, hay conjeturas razonables basadas en técnicas conocidas de teoría numérica; esto parecería cumplir con la definición de OP de 'heurística convincente'.
Tenga en cuenta también que los datos computacionales que hemos recopilado coinciden perfectamente con la predicción heurística. Así que creo que tenemos muy buenas razones para creer esta conjetura.
Este es un caso especial de la siguiente conjetura: Sea PAG ( X ) sea ​​un polinomio con coeficientes enteros. Entonces es primo infinitamente a menudo a menos que tenga una estúpida razón para no serlo. ("Razones estúpidas": PAG es reducible; los coeficientes no son coprimos; etc.) Esta conjetura general podría ser un mejor ejemplo de lo que OP está buscando.
Creo que la mayoría de los teóricos de números estarían extremadamente sorprendidos si esto no fuera primo infinitamente a menudo.

Existencia de un paralelepípedo rectangular con todas las aristas, todas las diagonales de las caras y siendo la diagonal principal números enteros.

Viabilidad de reformular todas las matemáticas en solo términos ultrafinitistas bien definidos

Desde el punto de vista de la física, hay algo extraño en la forma en que se usan las matemáticas. Según el principio de Church-Turing-Deutsch, todos los procesos físicos tienen descripciones computables (cuánticas), pero la forma en que hacemos matemáticas invoca conceptos no computables como los reales incontables. Lo que sucede si usamos las matemáticas en la práctica es que la incomputabilidad de cualquier concepto permanecerá siempre oculta en las partes intermedias, nunca surgirá en los resultados finales.

Esto sugiere que no es necesario invocar conceptos no computables en primer lugar, pero hasta ahora no ha habido mucho progreso por parte de los defensores del ultrafinitismo.

El problema del comportamiento asintótico de la cardinalidad máxima de un límite se establece en Z / 3 Z r como r hasta el infinito da lugar a la siguiente pregunta de sí/no que está abierta.

Un conjunto límite, aquí, es un conjunto sin tres puntos en una línea afín. Esto equivale a la existencia de X , y , z tal que X + y + z = 0 , o la existencia de una progresión aritmética de 3 términos (ver una respuesta relacionada).

¿La cardinalidad máxima de un tope se establece en Z / 3 Z r a O ( ( 3 d ) r ) para algunos d > 0 ?

Vea una publicación de blog de Terry Tao de hace un par de años donde expresa una opinión sobre el asunto pero también reconoce la disidencia de un buen amigo suyo.

Por ahora eso es realmente conocido. La respuesta es sí"

Hay algún axioma "natural" que sumado a ZFC decide CH? Interesante discusión en Introducción a la teoría de conjuntos, tercera edición, revisada y ampliada .

¿Qué es CH? (caracteres adicionales agregados para la longitud del comentario)
@DanielV, Hipótesis del Continuo.

La conjetura de Higman se refiere al número de clases de conjugación de tu T norte ( F q ) , el grupo de matrices triangulares superiores unipotentes con entradas en un campo finito con q elementos. La conjetura es que para un fijo norte el número de clases de conjugación de tu T norte ( F q ) viene dada por un polinomio en q . Esto ha sido probado hasta norte = 13 , pero más allá de eso es desconocido. La dificultad puede estar relacionada con el hecho de que tu T norte ( F q ) tiene un tipo de representación salvaje. Conozco una serie de intentos fallidos de prueba, y parece que la mayoría de las personas que piensan en esta conjetura creen que es cierta. Al mismo tiempo, hay una colección de subgrupos de tu T norte ( F q ) , conocidos como "grupos de patrones", para los cuales se sabe que una conjetura análoga es falsa.

Conjetura del divisor cero de Kaplansky


Dejar k ser un campo y GRAMO un grupo libre de torsión, entonces es el anillo de grupo k GRAMO ¿un dominio?

Todas las investigaciones hasta ahora han sido afirmativas. Este problema ha sido tratado en el libro " La estructura algebraica de los anillos de grupo" de D. Passman.

Es uno de los problemas más difíciles y menos abordables en todo el campo del álgebra.

Lo último que sé es que se ha probado para grupos solubles sin torsión. Todavía queda un largo camino por recorrer.

Otra pregunta natural después de esto (si lo anterior es cierto) es, si reemplazamos k por cualquier dominio D , decir Z

¿Cuál es la respuesta al problema en el que el número de Graham es un límite superior?

Citando a Wikipedia sobre la definición:

Conecte cada par de vértices geométricos de un hipercubo de n dimensiones para obtener un gráfico completo en 2n vértices. Colorea cada uno de los bordes de este gráfico de rojo o azul. ¿Cuál es el valor más pequeño de n para el cual cada coloración contiene al menos un subgrafo completo de un solo color en cuatro vértices coplanares?

Aquí hay algunos detalles sobre por qué es completamente desconocido:

  • Graham y Rotschild demostraron que 6 norte F ( F ( F ( F ( F ( F ( F ( 12 ) ) ) ) ) ) ) , dónde F ( X ) = 2 X 3 y denota notación de flecha hacia arriba.

  • Actualmente, el límite más conocido es: 13 norte < 2 ↑↑↑ 6 .

  • Los matemáticos pensaron que la respuesta era 6 , hasta el límite inferior de 11 fue probado. Ahora muchos no tienen idea de dónde esperar norte .

No creo que este problema sea un buen ejemplo. Hay una miríada de problemas combinatorios para los que se sabe que existe un límite, pero para los que no se conoce el mejor límite. El espíritu de esta publicación es más bien plantear un problema abierto con una respuesta de sí o no, para el cual la conjetura correcta no está clara o, peor aún, tiene evidencia de respaldo en ambos lados.

Realmente, realmente no tenemos idea de si la conjetura jacobiana es cierta.